Last visit was: 19 Nov 2025, 08:12 It is currently 19 Nov 2025, 08:12
Close
GMAT Club Daily Prep
Thank you for using the timer - this advanced tool can estimate your performance and suggest more practice questions. We have subscribed you to Daily Prep Questions via email.

Customized
for You

we will pick new questions that match your level based on your Timer History

Track
Your Progress

every week, we’ll send you an estimated GMAT score based on your performance

Practice
Pays

we will pick new questions that match your level based on your Timer History
Not interested in getting valuable practice questions and articles delivered to your email? No problem, unsubscribe here.
Close
Request Expert Reply
Confirm Cancel
User avatar
Marcab
Joined: 03 Feb 2011
Last visit: 22 Jan 2021
Posts: 850
Own Kudos:
4,853
 [57]
Given Kudos: 221
Status:Retaking after 7 years
Location: United States (NY)
Concentration: Finance, Economics
GMAT 1: 720 Q49 V39
GPA: 3.75
GMAT 1: 720 Q49 V39
Posts: 850
Kudos: 4,853
 [57]
5
Kudos
Add Kudos
51
Bookmarks
Bookmark this Post
Most Helpful Reply
User avatar
Bunuel
User avatar
Math Expert
Joined: 02 Sep 2009
Last visit: 19 Nov 2025
Posts: 105,389
Own Kudos:
Given Kudos: 99,977
Products:
Expert
Expert reply
Active GMAT Club Expert! Tag them with @ followed by their username for a faster response.
Posts: 105,389
Kudos: 778,264
 [23]
16
Kudos
Add Kudos
7
Bookmarks
Bookmark this Post
General Discussion
User avatar
BangOn
Joined: 27 Feb 2012
Last visit: 22 Mar 2019
Posts: 95
Own Kudos:
191
 [3]
Given Kudos: 22
Posts: 95
Kudos: 191
 [3]
2
Kudos
Add Kudos
1
Bookmarks
Bookmark this Post
User avatar
priyamne
Joined: 24 Apr 2012
Last visit: 15 Feb 2014
Posts: 36
Own Kudos:
54
 [1]
Given Kudos: 1
Posts: 36
Kudos: 54
 [1]
1
Kudos
Add Kudos
Bookmarks
Bookmark this Post
Ans:


both the statements alone don’t give any solution . When we combine them and add we get x^2+y^2=5a which is greater than 4a , but a can be zero( a is non-negative), in that case the answer becomes may be. Therefore the answer is (E).
avatar
Genfi
Joined: 08 Sep 2012
Last visit: 03 Jan 2014
Posts: 5
Own Kudos:
10
 [2]
Given Kudos: 28
Posts: 5
Kudos: 10
 [2]
2
Kudos
Add Kudos
Bookmarks
Bookmark this Post
Bunuel
Marcab
If a is non-negative, is x^2 + y^2 > 4a ?

(1) (x + y)^2 = 9a
(2) (x - y)^2 = a

Source: Jamboree
I am not convinced with the OA.

If a is non-negative, is x^2 + y^2 > 4a ?

(1) (x + y)^2 = 9a --> x^2+2xy+y^2=9a. Clearly insufficient.

(2) (x – y)^2 = a --> x^2-2xy+y^2=a. Clearly insufficient.

(1)+(2) Add them up 2(x^2+y^2)=10a --> x^2+y^2=5a. Also insufficient as x, y, and a could be 0 and x^2 + y^2 > 4a won't be true, as LHS and RHS would be in that case equal to zero. Not sufficient.

Answer: E.


Hi Bunel,

Can you please explain where am I going wrong:

(x^2+y^2) = x^2+2xy+y^2 = 9a..........(1)

x^2+y^2 >= 2xy ..........(2)

Substitute equation 2 in 1

Thus, (x^2+y^2+x^2+y^2) = 2(x^2+y^2) >= 9a

2(x^2+y^2) >= 9a

Finally, (x^2+y^2) >= 4.5a. Sufficient

(2) (x – y)^2 = a --> x^2-2xy+y^2=a. Clearly insufficient.

Answer: A
User avatar
Bunuel
User avatar
Math Expert
Joined: 02 Sep 2009
Last visit: 19 Nov 2025
Posts: 105,389
Own Kudos:
Given Kudos: 99,977
Products:
Expert
Expert reply
Active GMAT Club Expert! Tag them with @ followed by their username for a faster response.
Posts: 105,389
Kudos: 778,264
Kudos
Add Kudos
Bookmarks
Bookmark this Post
Genfi
Bunuel
Marcab
If a is non-negative, is x^2 + y^2 > 4a ?

(1) (x + y)^2 = 9a
(2) (x - y)^2 = a

Source: Jamboree
I am not convinced with the OA.

If a is non-negative, is x^2 + y^2 > 4a ?

(1) (x + y)^2 = 9a --> x^2+2xy+y^2=9a. Clearly insufficient.

(2) (x – y)^2 = a --> x^2-2xy+y^2=a. Clearly insufficient.

(1)+(2) Add them up 2(x^2+y^2)=10a --> x^2+y^2=5a. Also insufficient as x, y, and a could be 0 and x^2 + y^2 > 4a won't be true, as LHS and RHS would be in that case equal to zero. Not sufficient.

Answer: E.


Hi Bunel,

Can you please explain where am I going wrong:

(x^2+y^2) = x^2+2xy+y^2 = 9a..........(1)

x^2+y^2 >= 2xy ..........(2)

Substitute equation 2 in 1

Thus, (x^2+y^2+x^2+y^2) = 2(x^2+y^2) >= 9a

2(x^2+y^2) >= 9a

Finally, (x^2+y^2) >= 4.5a. Sufficient

(2) (x – y)^2 = a --> x^2-2xy+y^2=a. Clearly insufficient.

Answer: A

\(x^2+y^2\geq{4.5a}\) does NOT necessarily mean that \(x^2 + y^2 > 4a\). Consider x=y=a=0, in this case \(x^2 + y^2= 4a\).

Hope it's clear.
User avatar
madzstar
Joined: 24 Apr 2013
Last visit: 13 May 2014
Posts: 34
Own Kudos:
Given Kudos: 76
Schools: Duke '16
Schools: Duke '16
Posts: 34
Kudos: 23
Kudos
Add Kudos
Bookmarks
Bookmark this Post
Man i actually made a quick questimate and could tell within 15seconds that it would not work out. so answer I got was E. I have realised a lot of the gmat questions have the same concept and My question is though is this s dangerous way to appraoch the gmat exam
User avatar
arnabs
Joined: 06 Aug 2013
Last visit: 29 Oct 2020
Posts: 45
Own Kudos:
Given Kudos: 17
Posts: 45
Kudos: 15
Kudos
Add Kudos
Bookmarks
Bookmark this Post
Bunuel
Marcab
If a is non-negative, is x^2 + y^2 > 4a ?

(1) (x + y)^2 = 9a
(2) (x - y)^2 = a

Source: Jamboree
I am not convinced with the OA.

If a is non-negative, is x^2 + y^2 > 4a ?

(1) (x + y)^2 = 9a --> x^2+2xy+y^2=9a. Clearly insufficient.

(2) (x – y)^2 = a --> x^2-2xy+y^2=a. Clearly insufficient.

(1)+(2) Add them up 2(x^2+y^2)=10a --> x^2+y^2=5a. Also insufficient as x, y, and a could be 0 and x^2 + y^2 > 4a won't be true, as LHS and RHS would be in that case equal to zero. Not sufficient.

Answer: E.

Hi bunuel,
a clarification;
in inequalities that have multiple variables as is the case with this question and without any clear information about the variables i.e if they are negative, non-negative etc etc, the options are always insufficient, right?
the reason i am asking this question is because i approached this question the same way and got E.
nothing is mentioned about the variables and most importantly there is more than 1 variable. I mean the values for these variables could be anything. nothing is explicitly mentioned about them.
User avatar
Bunuel
User avatar
Math Expert
Joined: 02 Sep 2009
Last visit: 19 Nov 2025
Posts: 105,389
Own Kudos:
778,264
 [1]
Given Kudos: 99,977
Products:
Expert
Expert reply
Active GMAT Club Expert! Tag them with @ followed by their username for a faster response.
Posts: 105,389
Kudos: 778,264
 [1]
1
Kudos
Add Kudos
Bookmarks
Bookmark this Post
arnabs
Bunuel
Marcab
If a is non-negative, is x^2 + y^2 > 4a ?

(1) (x + y)^2 = 9a
(2) (x - y)^2 = a

Source: Jamboree
I am not convinced with the OA.

If a is non-negative, is x^2 + y^2 > 4a ?

(1) (x + y)^2 = 9a --> x^2+2xy+y^2=9a. Clearly insufficient.

(2) (x – y)^2 = a --> x^2-2xy+y^2=a. Clearly insufficient.

(1)+(2) Add them up 2(x^2+y^2)=10a --> x^2+y^2=5a. Also insufficient as x, y, and a could be 0 and x^2 + y^2 > 4a won't be true, as LHS and RHS would be in that case equal to zero. Not sufficient.

Answer: E.

Hi bunuel,
a clarification;
in inequalities that have multiple variables as is the case with this question and without any clear information about the variables i.e if they are negative, non-negative etc etc, the options are always insufficient, right?
the reason i am asking this question is because i approached this question the same way and got E.
nothing is mentioned about the variables and most importantly there is more than 1 variable. I mean the values for these variables could be anything. nothing is explicitly mentioned about them.

No, that's not correct. We do have some information about the variables: we know that a is non-negative, (x + y)^2 = 9a and (x - y)^2 = a. Yes, this info is not enough to answer whether x^2 + y^2 > 4a but if, for example, the question were whether \(x^2 + y^2 \geq{4a}\), then the answer would be C, not E.
avatar
hargun3045
Joined: 06 Feb 2018
Last visit: 29 Nov 2022
Posts: 16
Own Kudos:
Given Kudos: 5
Posts: 16
Kudos: 4
Kudos
Add Kudos
Bookmarks
Bookmark this Post
Hi Bunuel, thank you for the detailed explanation.
Another question,
If instead of a being non-negative, it was mentioned that 'a is positive' would the answer be A or C?
I may be wrong, but I suspected A similar to a deduction above i.e x^2+y^2 >= 4.5a
What are your thoughts?
avatar
HariharanIyeer0
Joined: 06 Jun 2017
Last visit: 11 Feb 2019
Posts: 7
Given Kudos: 45
Products:
Posts: 7
Kudos: 0
Kudos
Add Kudos
Bookmarks
Bookmark this Post
Bunuel
Marcab
If a is non-negative, is x^2 + y^2 > 4a ?

(1) (x + y)^2 = 9a
(2) (x - y)^2 = a

Source: Jamboree
I am not convinced with the OA.

If a is non-negative, is x^2 + y^2 > 4a ?

(1) (x + y)^2 = 9a --> x^2+2xy+y^2=9a. Clearly insufficient.

(2) (x – y)^2 = a --> x^2-2xy+y^2=a. Clearly insufficient.

(1)+(2) Add them up 2(x^2+y^2)=10a --> x^2+y^2=5a. Also insufficient as x, y, and a could be 0 and x^2 + y^2 > 4a won't be true, as LHS and RHS would be in that case equal to zero. Not sufficient.

Answer: E.


Would it be "C" , if the statement says a is a positive integer ?
User avatar
Bunuel
User avatar
Math Expert
Joined: 02 Sep 2009
Last visit: 19 Nov 2025
Posts: 105,389
Own Kudos:
Given Kudos: 99,977
Products:
Expert
Expert reply
Active GMAT Club Expert! Tag them with @ followed by their username for a faster response.
Posts: 105,389
Kudos: 778,264
Kudos
Add Kudos
Bookmarks
Bookmark this Post
HariharanIyeer0
Bunuel
Marcab
If a is non-negative, is x^2 + y^2 > 4a ?

(1) (x + y)^2 = 9a
(2) (x - y)^2 = a

Source: Jamboree
I am not convinced with the OA.

If a is non-negative, is x^2 + y^2 > 4a ?

(1) (x + y)^2 = 9a --> x^2+2xy+y^2=9a. Clearly insufficient.

(2) (x – y)^2 = a --> x^2-2xy+y^2=a. Clearly insufficient.

(1)+(2) Add them up 2(x^2+y^2)=10a --> x^2+y^2=5a. Also insufficient as x, y, and a could be 0 and x^2 + y^2 > 4a won't be true, as LHS and RHS would be in that case equal to zero. Not sufficient.

Answer: E.


Would it be "C" , if the statement says a is a positive integer ?

Yes, because in this case 5a will always be greater than 4a.
avatar
Shishou
Joined: 10 Jun 2019
Last visit: 08 Apr 2021
Posts: 103
Own Kudos:
Given Kudos: 112
Products:
Posts: 103
Kudos: 91
Kudos
Add Kudos
Bookmarks
Bookmark this Post
If a is non-negative, is x^2 + y^2 > 4a ?

(1) (x + y)^2 = 9a
(2) (x - y)^2 = a

Statement 1: when we expand we get x^2 + y^2 +2xy=9a; x^2 + y^2=9a-2xy.
The question stem becomes is 9a-2xy>4a? Or is 5a>2xy.All we know is that a is
Non negative.x and y can take any number of values. Insufficient

Statement 2 : when we expand we get x^2 + y^2- 2xy=a; x^2 + y^2=a+2xy.
The question stem becomes is a+2xy>4a? Or is 3a<2xy.Again,All we know is that a is
Non negative.x and y can take any number of values. Insufficient


Together, when we add statements 1 and 2,we get 2x^2 + 2y^2 =10a; This equals x^2 + y^2 =5a;
The question stem becomes is 5a>4a or is a>0.However,we know a is non negative meaning it could be positive or 0.Insufficient

Answer is E

Posted from my mobile device
User avatar
Jeeya
Joined: 02 Feb 2024
Last visit: 27 Aug 2025
Posts: 10
Given Kudos: 121
Posts: 10
Kudos: 0
Kudos
Add Kudos
Bookmarks
Bookmark this Post
Hi Bunuel,

I understand that options 1 and 2 are clearly not sufficient, but when we combine the two, we get:

(x+y)^2= 9(x-y)^2

taking their ratio: (x+y)^2/ (x-y)^2= 9/1

Upon taking square root on both sides, we get x+y=3 and x-y=1, which means x=2 and y=1. We can definitely say that x^2 + y^2 is greater than 4a. (since after this we know value of a=1).

Where am i going wrong here?

Bunuel
Marcab
If a is non-negative, is x^2 + y^2 > 4a ?

(1) (x + y)^2 = 9a
(2) (x - y)^2 = a

Source: Jamboree
I am not convinced with the OA.

If a is non-negative, is x^2 + y^2 > 4a ?

(1) (x + y)^2 = 9a --> x^2+2xy+y^2=9a. Clearly insufficient.

(2) (x – y)^2 = a --> x^2-2xy+y^2=a. Clearly insufficient.

(1)+(2) Add them up 2(x^2+y^2)=10a --> x^2+y^2=5a. Also insufficient as x, y, and a could be 0 and x^2 + y^2 > 4a won't be true, as LHS and RHS would be in that case equal to zero. Not sufficient.

Answer: E.
User avatar
Bunuel
User avatar
Math Expert
Joined: 02 Sep 2009
Last visit: 19 Nov 2025
Posts: 105,389
Own Kudos:
Given Kudos: 99,977
Products:
Expert
Expert reply
Active GMAT Club Expert! Tag them with @ followed by their username for a faster response.
Posts: 105,389
Kudos: 778,264
Kudos
Add Kudos
Bookmarks
Bookmark this Post
Jeeya
Hi Bunuel,

I understand that options 1 and 2 are clearly not sufficient, but when we combine the two, we get:

(x+y)^2= 9(x-y)^2

taking their ratio: (x+y)^2/ (x-y)^2= 9/1

Upon taking square root on both sides, we get x+y=3 and x-y=1, which means x=2 and y=1. We can definitely say that x^2 + y^2 is greater than 4a. (since after this we know value of a=1).

Where am i going wrong here?

Bunuel
Marcab
If a is non-negative, is x^2 + y^2 > 4a ?

(1) (x + y)^2 = 9a
(2) (x - y)^2 = a

Source: Jamboree
I am not convinced with the OA.

If a is non-negative, is x^2 + y^2 > 4a ?

(1) (x + y)^2 = 9a --> x^2+2xy+y^2=9a. Clearly insufficient.

(2) (x – y)^2 = a --> x^2-2xy+y^2=a. Clearly insufficient.

(1)+(2) Add them up 2(x^2+y^2)=10a --> x^2+y^2=5a. Also insufficient as x, y, and a could be 0 and x^2 + y^2 > 4a won't be true, as LHS and RHS would be in that case equal to zero. Not sufficient.

Answer: E.

Read the solution carefully. x, y, and a could be 0.

First, you can’t divide both sides of (x + y)^2 = 9(x - y)^2 by (x - y)^2 unless you know for sure that (x - y) ≠ 0, which you don’t.

Second, when you take the square root of both sides, you get |x + y| = 3|x - y|, not (x + y) = 3(x - y).

Even if you assume (x + y) = 3(x - y), that doesn’t mean x + y = 3 and x - y = 1. It could just as well be x + y = 9 and x - y = 3. That equation only implies x = 2y, which still doesn't give specific values.

And finally, pure algebraic questions are no longer a part of the DS syllabus of the GMAT.

DS questions in GMAT Focus encompass various types of word problems, such as:

  • Word Problems
  • Work Problems
  • Distance Problems
  • Mixture Problems
  • Percent and Interest Problems
  • Overlapping Sets Problems
  • Statistics Problems
  • Combination and Probability Problems

While these questions may involve or necessitate knowledge of algebra, arithmetic, inequalities, etc., they will always be presented in the form of word problems. You won’t encounter pure "algebra" questions like, "Is x > y?" or "A positive integer n has two prime factors..."

Check GMAT Syllabus for Focus Edition

You can also visit the Data Sufficiency forum and filter questions by OG 2024-2025, GMAT Prep (Focus), and Data Insights Review 2024-2025 sources to see the types of questions currently tested on the GMAT.

So, you can ignore this question.

Hope it helps.­
Moderators:
Math Expert
105389 posts
496 posts